Difference between revisions of "2007 AIME I Problems/Problem 11"
Lilcritters (talk | contribs) (→Solution) |
Firebolt360 (talk | contribs) (→Solution) |
||
Line 7: | Line 7: | ||
Now consider the range of numbers such that <math>b(p)=45</math>. These numbers are <math>\left\lceil\frac{44^2 + 45^2}{2}\right\rceil = 1981</math> to <math>2007</math>. There are <math>2007 - 1981 + 1 = 27</math> (1 to be inclusive) of them. <math>27*45=1215</math>, and <math>215+740= | Now consider the range of numbers such that <math>b(p)=45</math>. These numbers are <math>\left\lceil\frac{44^2 + 45^2}{2}\right\rceil = 1981</math> to <math>2007</math>. There are <math>2007 - 1981 + 1 = 27</math> (1 to be inclusive) of them. <math>27*45=1215</math>, and <math>215+740= | ||
\boxed{955}</math>, the answer. | \boxed{955}</math>, the answer. | ||
+ | |||
+ | ==Solution 2== | ||
+ | Let <math>p</math> be in the range of <math>a^2 \le p < (a+1)^2</math> TBC | ||
== See also == | == See also == |
Revision as of 11:14, 19 November 2020
Contents
Problem
For each positive integer , let denote the unique positive integer such that . For example, and . If find the remainder when is divided by 1000.
Solution
and . Therefore if and only if is in this range, or . There are numbers in this range, so the sum of over this range is . , so all numbers to have their full range. Summing this up with the formula for the sum of the first squares (), we get . We need only consider the because we are working with modulo .
Now consider the range of numbers such that . These numbers are to . There are (1 to be inclusive) of them. , and , the answer.
Solution 2
Let be in the range of TBC
See also
2007 AIME I (Problems • Answer Key • Resources) | ||
Preceded by Problem 10 |
Followed by Problem 12 | |
1 • 2 • 3 • 4 • 5 • 6 • 7 • 8 • 9 • 10 • 11 • 12 • 13 • 14 • 15 | ||
All AIME Problems and Solutions |
The problems on this page are copyrighted by the Mathematical Association of America's American Mathematics Competitions.